Pagina 1 di 2

IMO 1984 n°1

Inviato: 20 gen 2011, 13:26
da amatrix92
Dimostrare che $ 0 \leq yz + zx + xy − 2xyz \leq \frac{7}{27} $
$ 27 $ , dove $ x, y, z $ sono reali
positivi che soddisfano $ x + y + z = 1 $.

Re: IMO 1984 n°1

Inviato: 20 gen 2011, 16:24
da staffo
Divido ovviamente l'uguaglianza in due e le risolvo singolarmente.

1) $ yz + zx + xy − 2xyz\geq 0 $
WLOG (si scrive così, sto cercando di imparare l'inglese xd) pongo $ x\geq y\geq z $;
raccolgo un $ xy $ e ottengo: $ yz + zx + xy (1-2z) \geq 0 $;

so che $ x\geq z $ per cui $ z $ è, al più, uguale a $ \frac {1}{2} $
da cui ne consegue che la parentesi non sarà mai negative ed, essendo tutti gli altri termini positivi, la disuguaglianza uno è verificata.


2)$ yz + zx + xy - 2xyz \leq \frac{7}{27} $;
Trovo il valore per cui risulta l'ugluaglianza, cioè $ x=y=z=\frac{1}{3} $ (perchè ho voglia);

Qui è il punto critico su cui ho perso tutto il tempo:devo massimizzare la parte snistra della disuguaglianza: se sarà vera per quella, sarà vera per tutti.
Come massimizzarla?

Il problema è che per massimizzare $ xy + yz + zx $ dovrei porre $ x=y=z=\frac{1}{3} $, invece per massimizzare $ -2xyz $ dovrei porre uno tra i tre nulli.
Ovviamente le due cose non possono coesistere e devo trovare un compromesso. Potrei dire che, essendo $ x,y,z \leq 1 $, $ yz + zx + xy $ incidono di più (in quanto prodotti a due a due e non a tre a tre) di $ -2xyz $ e quindi massimizzo quelli, da cui ottengo $ \frac{7}{27}\leq\frac{7}{27} $ che è vera.

Volevo sapere se era ammesso un ragionamento del genere e, in alternativa, se potevo usare qualche teorema o risultato a riguardo.

Re: IMO 1984 n°1

Inviato: 20 gen 2011, 16:33
da Mist
amatrix92 ha scritto:Dimostrare che $ 0 \leq yz + zx + xy − 2xyz \leq \frac{7}{27} $
$ 27 $ , dove $ x, y, z $ sono reali
positivi che soddisfano $ x + y + z = 1 $.
Bon, proviamoci...
$yz + zx + xy − 2xyz = (yz + zx + xy)\cdot 1 − 2xyz = (yz + zx + xy)\cdot (x+y+z) − 2xyz = xyz+yz^2 + zy^2+xy^2 + xz^2 + yx^2 $ $+zx^2 = xyz+(x+y)z^2+(z+x)y^2+(y+z)x^2 = xyz +(1-z)z^2+(1-y)y^2+(1-x)x^2 \geq 0$

P.S. ho visto che staffo mi ha anticipato, ma non mi sembra molto corretto il suo ragionamento... Ovvero, non mi sembra che dimostri realmente che quel coso là è maggiore di zero... alla seconda parte devo pensarci, posto la prima così per confrontalra con staffo

P.P.S: WLOG = without loss of generality

Re: IMO 1984 n°1

Inviato: 20 gen 2011, 16:37
da paga92aren
staffo ha scritto: Qui è il punto critico su cui ho perso tutto il tempo:devo massimizzare la parte snistra della disuguaglianza: se sarà vera per quella, sarà vera per tutti.
Come massimizzarla?

Il problema è che per massimizzare $ xy + yz + zx $ dovrei porre $ x=y=z=\frac{1}{3} $, invece per massimizzare $ -2xyz $ dovrei porre uno tra i tre nulli.
Ovviamente le due cose non possono coesistere e devo trovare un compromesso. Potrei dire che, essendo $ x,y,z \leq 0 $, $ yz + zx + xy $ incidono di più (in quanto prodotti a due a due e non a tre a tre) di $ -2xyz $ e quindi massimizzo quelli, da cui ottengo $ \frac{7}{27}\leq\frac{7}{27} $ che è vera.

Volevo sapere se era ammesso un ragionamento del genere e, in alternativa, se potevo usare qualche teorema o risultato a riguardo.
Credo che un ragionamento del genere non sia ammesso se non lo dimostri in maniera adeguata.
Perché i prodotti due a due incidono di più di quelli tre a tre? se prendo numeri grandi non è vero (per grandi basta maggiori di 3)
Inoltre anche se incidono di più non significa che il massimo si ha in quel valore (un terzo) ma che il massimo è "più vicino" a un terzo che a zero.

(a un certo punto scrivi x,y,z<0 e poi nel punto 1 $z\leq \frac{1}{3}$ quindi anche di 1/2)

Re: IMO 1984 n°1

Inviato: 20 gen 2011, 16:43
da staffo
il primo ragionamento non credo faccia una piega, non credo possa essere contestabile, anzi, più chiaro di così....

per quel fatto là, infatti io ho detto che incidono di più poichè $ x,y,z \leq 1 $ EDIT (avevo scritto erroneamente 0)
questo mi porta a dire che devo massimizzare quelli per ottenere il massimo, e so che non è una dimostrazione rigorosa, e infatti volevo sapere se qualcuno sapeva per caso un teorema a riguardo o riusciva a dimostrarlo (perchè che sia così è evidente)

Re: IMO 1984 n°1

Inviato: 20 gen 2011, 16:46
da Mist
Perdonami, mi ero perso il WLOG mentre leggevo :D è a posto così, mi ero scordato mentre leggevo ciò che avevi scritto prima (mio dio :shock: )

Re: IMO 1984 n°1

Inviato: 20 gen 2011, 17:40
da amatrix92
Staffo: la prima parte anche se non è bellissima può andare. La seconda mi pare che è poco chiara la massimizzazione
Mist: prima parte perfetta uguale alla mia solzuione, però potevi fermarti al terzo rigo, gli altri sono superflui; al terzo rigo hai una somma di termini tutti positivi.

Re: IMO 1984 n°1

Inviato: 20 gen 2011, 17:44
da staffo
Perchè dici che la pria parte non è blla? è semplificativa al massimo e ti evita un sacco di calcoli inutili.

Per la massimizzazione, ripeto, non è che è poco chiara, è che non è rigorosamente dimostrata, che è quello che vorrei fare (e che non mi viene) se a qualcuno venisse la massimizzazione che spiegasse ciò che ho fatto è ben lieto il suo apporto =)

Re: IMO 1984 n°1

Inviato: 20 gen 2011, 17:48
da amatrix92
Non è bella perchè necessita di un ordinamento mentre quella di mist è più generica. Però una soluzione è tale, bella o non bella.

Edit: Saffo: come fai a dire che $ yz+zx+xy $ si massimizza per $ x=y=z=\frac{1}{3} $ ?

Re: IMO 1984 n°1

Inviato: 20 gen 2011, 18:32
da Euler
Per il secondo punto si può usare il bunching:
Sviluppo la disuguaglianza fino a ottenere $\displaystyle \sum_{sym}x^2y +xyz\leq \frac{7}{27}$
Ora scrivo $\frac{7}{27}$ come $\frac{7(x+y+z)^3}{27}$, quindi:
$\displaystyle 7(x^3+y^3+z^3)+21\sum_{sym}x^2y+9xyz\geq 27\sum_{sym}x^2y+27xyz$
e adesso è il momento del bunching, permettendo delle semplificazioni:
$\displaystyle x^3+y^3+z^3+9xyz\geq 3\sum_{sym} x^2y+27xyz$
$\displaystyle (x+y+z)^3\geq 27xyz$
la quale è vera per AM-GM $\square$

Re: IMO 1984 n°1

Inviato: 20 gen 2011, 18:40
da paga92aren
Per massimizzare l'espressione $yz + zx + xy − 2xyz \leq \frac{7}{27}$impongo WLOG $x\geq y\geq z$ e $z\leq \frac{1}{3}$ e lo sostituisco nell'ultimo termine:
$yz + zx + xy − \frac{2}{3}xy=z(x+y)+\frac{1}{3}xy$
Per AM-GM ho che $(\frac{x+y}{2})^2\geq xy$ quindi LHS$\leq z(1-z)+\frac{1}{12}(1-z)^2\leq\frac{7}{27}$ per ogni $0 \leq z \leq \frac{1}{3}$ chiamo $1-z=a$ e ottengo $-\frac{11}{12}a^2+a\geq \frac{7}{27}$ con $\frac{2}{3}\leq a \leq 1$.
Quella è l'equazione di una parabola col vertice in $a=\frac{6}{11}$ quindi il suo massimo nell'intervallo ($\frac{2}{3}, 1$) è in $\frac{2}{3}$ ed è $\frac{7}{27}$ da cui la tesi

Re: IMO 1984 n°1

Inviato: 20 gen 2011, 19:36
da patatone
@euler:credo che tu abbia sbagliato i calcoli, il bunching non ti basta, ma potresti usare schur...

@paga:anche qui mi pare ci sia un errore..
$-2xyz<=-2/3xy$ implica $z>=1/3$

Re: IMO 1984 n°1

Inviato: 20 gen 2011, 19:53
da paga92aren
patatone ha scritto:@paga:anche qui mi pare ci sia un errore..
$-2xyz<=-2/3xy$ implica $z>=1/3$
:oops: è vero, allora uso $\sum xy\leq \frac{(\sum x)^2}{3}=\frac{1}{3}$ (che lascio da dimostrare a chi non la conosce) e poi rimane da dimostrare che $\frac{1}{27}\leq xyz$

Re: IMO 1984 n°1

Inviato: 20 gen 2011, 20:02
da Mist
"... e allora arrivò patatone ad illuminarci il cammino"

bon, un prodotto di numeir reali si massimizza quando tutti i fattori sono uguali tra loro e quindi hai finito, perchè la cosa si riduce a dire che $x \ge \frac{1}{3}$ che è vero: bravo paga !

Re: IMO 1984 n°1

Inviato: 20 gen 2011, 21:05
da patatone
mi sa tanto che ancora qualcosa non quadra